Help Solving a differential equation

Click For Summary
To solve the differential equation (x+2y)y' = y, the discussion suggests using substitution, specifically v = (x + 2y), but notes that this does not yield a separable or first-order linear equation. Participants recommend rewriting the equation in the form M(x,y)dx + N(x,y)dy = 0 and checking for exactness, with the possibility of finding an integrating factor if it is not exact. Another approach mentioned is to express the equation as y(dx/dy) = x + 2y, which forms a linear equation for x as a function of y. The conversation emphasizes that multiple methods exist for solving the equation, highlighting the flexibility in approaches.
Giuseppe
Messages
42
Reaction score
0
Hello, i was wondering if someone can help me along with solving this differential equation.(x+2y)y'=y

I believe you use substitution. Right now I am setting my substitution to
v=(x+2y), but then when i follow through with my work, it doesn't simplify down to a seperable or first order linear equation. Am I doing something wrong?

-giuseppe
 
Last edited:
Physics news on Phys.org
try this should make a nice linear in terms of x

divide the left side by y

then divide 1 by y' and you should end up with a linear in terms of x
 
Rewrite this in the form M(x,y)dx + N(x,y)dy = 0. Then, check if this is an exact differential equation. If not, can you find an integrating factor to make it exact?

I don't think substitution works here. It's just not separable.
 
This is NOT an exact equation either.

I like Valhalla's suggestion. You can rewrite it as

y(dx/dy)= x+ 2y which is a LINEAR equation for x as a function of y. If you really need y as a function of x, invert the function.
 
I didn't say it was exact... I said he should check if it was exact. :smile:

It's easy enough to make it exact with an integrating factor though.

There is more than one way to skin a cat...
 
Question: A clock's minute hand has length 4 and its hour hand has length 3. What is the distance between the tips at the moment when it is increasing most rapidly?(Putnam Exam Question) Answer: Making assumption that both the hands moves at constant angular velocities, the answer is ## \sqrt{7} .## But don't you think this assumption is somewhat doubtful and wrong?

Similar threads

  • · Replies 10 ·
Replies
10
Views
2K
Replies
10
Views
2K
  • · Replies 3 ·
Replies
3
Views
882
  • · Replies 7 ·
Replies
7
Views
2K
  • · Replies 8 ·
Replies
8
Views
1K
  • · Replies 18 ·
Replies
18
Views
3K
  • · Replies 4 ·
Replies
4
Views
2K
Replies
7
Views
2K
  • · Replies 5 ·
Replies
5
Views
2K
Replies
3
Views
2K